Difference between revisions of "2003 AMC 12A Problems/Problem 23"

(Created page with '== Solution == Using logarithmic rules, we see that <cmath>\log_{a}a-\log_{a}b+\log_{b}b-\log_{b}a = 2-(\log_{a}b+\log_{b}a</cmath> <math>=2-(\log_{a}b+\frac {1}{\log_{a}b}</ma…')
 
(Solution)
Line 4: Line 4:
  
 
<cmath>\log_{a}a-\log_{a}b+\log_{b}b-\log_{b}a = 2-(\log_{a}b+\log_{b}a</cmath>
 
<cmath>\log_{a}a-\log_{a}b+\log_{b}b-\log_{b}a = 2-(\log_{a}b+\log_{b}a</cmath>
<math>=2-(\log_{a}b+\frac {1}{\log_{a}b}</math><math>
+
<cmath>=2-(\log_{a}b+\frac {1}{\log_{a}b}</cmath>
  
Since </math>a<math> and </math>b<math> are both positive, using [[AM-GM]] gives that the term in parentheses must be at least two, so the largest possible values is </math>2-2=\boxed{0}.$
+
Since <math>a</math> and <math>b</math> are both positive, using [[AM-GM]] gives that the term in parentheses must be at least two, so the largest possible values is <math>2-2=\boxed{0}.</math>

Revision as of 18:33, 22 February 2010

Solution

Using logarithmic rules, we see that

\[\log_{a}a-\log_{a}b+\log_{b}b-\log_{b}a = 2-(\log_{a}b+\log_{b}a\] \[=2-(\log_{a}b+\frac {1}{\log_{a}b}\]

Since $a$ and $b$ are both positive, using AM-GM gives that the term in parentheses must be at least two, so the largest possible values is $2-2=\boxed{0}.$